Jump to content

lahantaithe99's Content

There have been 878 items by lahantaithe99 (Search limited from 06-06-2020)



Sort by                Order  

#491414 $\boxed{\text{Chuyên Đề}}$ Bất đẳng thức - Cực trị

Posted by lahantaithe99 on 08-04-2014 - 17:11 in Bất đẳng thức và cực trị

Bài 150: CMR với mọi số thực dương $a,b,c$ ta luôn có:

$\sqrt{\frac{a^2+2b^2}{a^2+ab+bc}}+\sqrt{\frac{b^2+2c^2}{b^2+bc+ca}}+\sqrt{\frac{c^2+2a^2}{c^2+ca+ab}}\geq 3$

 

150

 

Áp dụng BĐT Cô si

 

$\sum \sqrt{\frac{a^2+2b^2}{a^2+ab+bc}}\geqslant 3\sqrt[6]{\frac{(a^2+2b^2)(b^2+2c^2)(c^2+2a^2)}{(a^2+ab+bc)(b^2+bc+ac)(c^2+ac+ab)}}$ $(1)$

 

Theo BĐT Bunhiacopxki

 

$(a^2+b^2+b^2)(a^2+c^2+a^2)\geqslant (a^2+ab+bc)^2$

 

Thiết lập tương tự với các số còn lại và rút gọn ta có

 

$(a+2b^2)(b^2+2c^2)(c^2+2a^2)\geqslant (a^2+bc+ab)(b^2+bc+ac)(c^2+ac+ab)$ $(2)$ 

 

Kết hợp $(1)$ với $(2)$ ta có đpcm




#491415 $\boxed{\text{Chuyên Đề}}$ Bất đẳng thức - Cực trị

Posted by lahantaithe99 on 08-04-2014 - 17:12 in Bất đẳng thức và cực trị

Lahan ơi, mình tưởng chỗ màu đỏ kia phải là $(a+b)^2(c+1)^2$

Ừ, sorry bạn




#489682 $\boxed{\text{Chuyên Đề}}$ Bất đẳng thức - Cực trị

Posted by lahantaithe99 on 30-03-2014 - 19:04 in Bất đẳng thức và cực trị

  $a,b > 0$  thỏa   mãn    $3a^{2}  + 2b^{2} \leq  5$

Tìm min     $ \frac{a}{2}+ b+  \frac{2}{ab}$

:nav:  :nav:  :nav:

Áp dụng BĐT Cô si 

 

$5=3a^2+2b^2=a^2+a^2+a^2+b^2+b^2\geqslant 5.\sqrt[5]{a^6b^4}\Rightarrow a^6b^4\leqslant 1\Rightarrow a^3b^2\leqslant 1$

 

Tiếp tục áp dụng BĐT cô si và sử dụng $a^3b^2\leqslant 1$

 

$\frac{a}{2}+b+\frac{2}{ab}=\frac{a}{2}+\frac{b}{2}+\frac{b}{2}+\frac{1}{2ab}+\frac{1}{2ab}+\frac{1}{2ab}+\frac{1}{2ab}$

 

$\geqslant 7.\sqrt[7]{\frac{1}{2^7a^3b^2}}\geqslant \frac{7}{2}$

 

@Viet Hoang 99: Đừng sửa trích dẫn của người khác nhé.




#488588 $\boxed{\text{Chuyên Đề}}$ Bất đẳng thức - Cực trị

Posted by lahantaithe99 on 24-03-2014 - 19:53 in Bất đẳng thức và cực trị

TOPIC yêu cầu bài nào đã làm xong được tô màu đỏ  :angry: 

 

119) Cho tam giác $ABC$ có diện tích là $\frac{3}{2}$. CMR: $(\frac{1}{a}+\frac{1}{b}+\frac{1}{c})(\frac{1}{h_{a}}+\frac{1}{h_{b}}+\frac{1}{h_{c}})\geq 3$

 

 

 

 

119 Ta có 

 

$a.h_{a}=b.h_{b}=c.h_{c}=3\Rightarrow a=\frac{h_{a}}{3};b=\frac{h_{b}}{3};c=\frac{h_{c}}{3}$

 

Khi đó

 

$(\sum \frac{1}{a})(\sum\frac{1}{h_{a}} )=(\sum \frac{h_{a}}{3})(\sum \frac{1}{h_{a}})\geqslant (\frac{3}{\sqrt{3}})^2=3$

 

(đúng theo bđt Bunhiacopxki




#488063 $\boxed{\text{Chuyên Đề}}$ Bất đẳng thức - Cực trị

Posted by lahantaithe99 on 21-03-2014 - 12:38 in Bất đẳng thức và cực trị

Cho 3 số dương a, b, c thoả mãn a + b + c = 1

a, CMR $\frac{a + bc}{b + c}$  +  $\frac{b + ca}{c + a}$  +  $\frac{c + ab}{a + b}$    $\geq$   2

b, Tìm GTNN của P = $\frac{a}{ab + c}$  +  $\frac{b}{bc + a}$ + $\frac{c}{ac + b}$

 

P/s:Bài này sử dụng phương pháp đổi biến nhé!!!

a)

 

Ta có $\sum \frac{a+bc}{b+c}=\sum \frac{a(a+b+c)+bc}{b+c}=\sum \frac{(a+b)(a+c)}{b+c}$

 

Cô si từng cặp một ta có 

 

$\sum \frac{(a+b)(a+c)}{b+c}\geqslant 2(a+b+c)=2$

 

b) 

Tương tự phần a ta có

 

$\sum \frac{a}{ab+c}=\sum \frac{a}{(c+a)(c+b)}$

 

Áp dụng bđt Cauchy

 

$\frac{a}{(a+c)(c+b)}+\frac{27a(b+c)}{8}+\frac{27a(a+c)}{8}\geqslant \frac{27}{4}a$

 

Thiết lập tương tự ta có

 

$\sum \frac{a}{(a+c)(c+b)}+\frac{27}{8}[(a+b+c)^2+ab+bc+ac]\geqslant \frac{27}{4}\sum a$

 

$\Leftrightarrow \sum \frac{a}{(a+c)(c+b)}\geqslant \frac{27}{4}-\frac{27}{8}-\frac{27}{8}(ab+bc+ac)$

 

Lại có $ab+bc+ac\leqslant \frac{1}{3}$

 

$\Rightarrow \sum \frac{a}{(b+c)(a+c)}\geqslant \frac{9}{4}$




#492301 $\boxed{\text{Chuyên Đề}}$ Bất đẳng thức - Cực trị

Posted by lahantaithe99 on 11-04-2014 - 22:12 in Bất đẳng thức và cực trị

Bài 140.  Cho $a,b,c,d,e$ là các số thực không âm biết $a+b+c+d+e=5$. Chứng minh : $abc+bcd+cde+dea+eab\leq 5$

Ở đây này

http://diendantoanho...từ-mathlinksro/




#493344 $\boxed{\text{Chuyên Đề}}$ Bất đẳng thức - Cực trị

Posted by lahantaithe99 on 16-04-2014 - 20:06 in Bất đẳng thức và cực trị

Bài143. Cho các số thực dương $x,y,z$ thỏa mãn : $xyz=x+y+z+2$

Chứng minh rằng : $\frac{1}{\sqrt{xy}}+\frac{1}{\sqrt{yz}}+\frac{1}{xz}\leq \frac{3}{2}$

 

Cái chỗ $xz$ kia chắc là $\sqrt{xz}$

 

Từ giả thiết thu được $\frac{x}{x+1}+\frac{y}{y+1}+\frac{z}{z+1}=2$ (cái này tính toán thử ra bao nhiêu TH mới phát hiện ra)

 

Áp dụng BĐT Bunhiacopxki ta có

 

$(\frac{1}{\sqrt{xy}}+\frac{1}{\sqrt{yz}}+\frac{1}{\sqrt{xz}})^2\leqslant (\frac{x}{x+1}+\frac{y}{y+1}+\frac{z}{z+1})(\frac{x+1}{xyz}+\frac{y+1}{xyz}+\frac{z+1}{xyz})$

 

$=\frac{2(x+y+z+3)}{xyz}=\frac{2(xyz+1)}{xyz}=2+\frac{2}{xyz}$

 

Từ giả thiết ta dễ chứng minh $xyz\geqslant 8\Rightarrow 2+\frac{2}{xyz}\leqslant \frac{9}{4}$ 

 

Do đó $(\sum \frac{1}{\sqrt{xy}})^2\leqslant \frac{9}{4}\Rightarrow \sum \frac{1}{\sqrt{xy}}\leqslant \frac{3}{2}$




#510633 $\boxed{\text{Chuyên Đề}}$ Bất đẳng thức - Cực trị

Posted by lahantaithe99 on 04-07-2014 - 01:06 in Bất đẳng thức và cực trị

Bài183:Cho a,b,c >0 và a+b+c=1.CMR $ab+bc+ca\geq 8(a^2+b^2+c^2)(a^2b^2+b^2c^2+c^2a^2)$

Bài 184:Cho x>0 .CMR: $\frac{2\sqrt{2}}{\sqrt{x+1}}+\sqrt{x}\leq \sqrt{x+9}$

 

Bài 186:Cho a,b >0 CMR :$\frac{2a^2+3b^2}{2a^3+3b^3}+\frac{2b^2+3a^2}{2b^3+3a^3}\leq \frac{4}{a+b}$

 

Bài 183: Xem lại đề bạn nhé hình như có vấn đề rồi !

 

Bài 184. Áp dụng BĐT Bunhiacopxki:

 

$(\frac{2\sqrt{2}}{\sqrt{x+1}}+\sqrt{x})^2\leqslant (\frac{8}{x+1}+1)(1+x)=x+9$

 

$\Rightarrow \sqrt{\frac{8}{x+1}}+\sqrt{x}\leqslant \sqrt{x+9}$ (đpcm)

 

Bài 186: 

 

Áp dụng BĐT Bunhiacopxki, ta giảm bậc cho BĐT đỡ cồng kềnh

 

$(2a^3+3b^3)(2a+3b)\geqslant (2a^2+3b^2)^2\Rightarrow \frac{2a^2+3b^2}{2a^3+3b^3}\leqslant \frac{2a+3b}{2a^2+3b^2}$

 

Do đó ta đi chứng minh

 

 $\sum \frac{2a+3b}{2a^2+3b^2}\leqslant \frac{4}{a+b}\Leftrightarrow \sum (\frac{2a+3b}{2a^2+3b^2}-\frac{2}{a+b})\leqslant 0$

 

$\Leftrightarrow -(a-b)^2\left [ \frac{12(a^2+b^2)-ab}{(a+b)(2a^2+3b^2)(2b^2+3a^2)} \right ]\leqslant 0$ (luôn đúng) 

 

Do đó ta có đpcm




#504525 $\boxed{\text{Chuyên Đề}}$ Bất đẳng thức - Cực trị

Posted by lahantaithe99 on 06-06-2014 - 19:32 in Bất đẳng thức và cực trị

 

BÀI 176:

Cho a,b,c>0 thỏa $a^{2}+b^{2}+c^{2}=1$ . CM

$\frac{1}{1-bc}+\frac{1}{1-ac}+\frac{1}{1-ab}\leq \frac{9}{2}$

 

BĐT cần chứng minh $\Leftrightarrow \sum \frac{2bc}{2-2bc}\leqslant \frac{3}{2}$

 

Ta có 

 

$\sum \frac{2bc}{2-2bc}\leqslant \sum \frac{2bc}{2-(b^2+c^2)}=\sum \frac{2bc}{(a^2+b^2)+(a^2+c^2)}$

 

$=\frac{1}{2}\sum \frac{4bc}{(a^2+b^2)+(a^2+c^2)}\leqslant \frac{1}{2}\sum \frac{(b+c)^2}{(a^2+b^2)+(a^2+c^2)}$

 

$\leqslant \frac{1}{2}\sum (\frac{b^2}{a^2+b^2}+\frac{c^2}{a^2+c^2})=\frac{3}{2}$

 

Nên ta có đpcm




#503190 $\boxed{\text{Chuyên Đề}}$ Bất đẳng thức - Cực trị

Posted by lahantaithe99 on 01-06-2014 - 10:20 in Bất đẳng thức và cực trị

Bài 169: Với $a,b,c>0$ chứng minh rằng:

$P=\sum \frac{a^3c}{b^2(2c^2+ab)}\geq 1$

Bài 170: Với $a,b,c$ là những số thực dương, chứng minh rằng 

$P=\sum \frac{a^3b}{ab^2+1}\geq \frac{abc(a+b+c)}{1+abc}$

 

169

Ta có

 

$P=\sum \frac{\frac{a^2}{b^2}}{\frac{2c^2+ab}{ac}}\geqslant \frac{(\frac{a}{b}+\frac{b}{c}+\frac{c}{a})^2}{3(\frac{a}{b}+\frac{b}{c}+\frac{c}{a})}=\frac{1}{3}(\frac{a}{b}+\frac{b}{c}+\frac{c}{a})\geqslant 1$

 

Đẳng thức xảy ra tại $a=b=c>0$

 

170

 

$P=\sum \frac{a^2}{a+\frac{1}{ab}}\geqslant \frac{(a+b+c)^2}{a+b+c+\frac{a+b+c}{abc}}=\frac{abc(a+b+c)}{1+abc}$

 

(đpcm)




#493791 $\boxed{\text{Chuyên Đề}}$ Bất đẳng thức - Cực trị

Posted by lahantaithe99 on 18-04-2014 - 21:44 in Bất đẳng thức và cực trị

146

a,b,c là độ dài 3 cạnh tam giác CMR

$(a^{2}b+b^{2}c+c^{2}a)^{2}\geq abc(a+b+c)(a^{2}+b^{2}+c^{2})$

Áp dụng BĐT Bunhiacopxki ta có

 

$(a^2b+b^2c+c^2a)(\frac{1}{a}+\frac{1}{b}+\frac{1}{c})\geqslant (a+b+c)^2$

 

$\Rightarrow a^2b+b^2c+c^2a\geqslant \frac{abc(a+b+c)^2}{ab+bc+ac}$

 

$\Leftrightarrow (a^2b+b^2c+c^2a)^2\geqslant \frac{abc(a+b+c)^2(a^2b+b^2c+c^2a)}{ab+bc+ac}$

 

Để kết thúc bài toán ta cần chứng minh

 

$(a+b+c)(a^2b+b^2c+c^2a)\geqslant (ab+bc+ac)(a^2+b^2+c^2)$

 

$a^2b^2+b^2c^2+c^2a^2\geqslant abc(a+b+c)$ (BĐT này đúng theo $AM-GM$)




#487948 $\boxed{\text{Chuyên Đề}}$ Bất đẳng thức - Cực trị

Posted by lahantaithe99 on 20-03-2014 - 17:03 in Bất đẳng thức và cực trị

 

Bài 111: Cho $a,b,c$ là các số thực dương. CMR:

$\sum \frac{a}{(b+c)^2}\geq \frac{9}{4(\sum a)}$

 

P/s: HẾT!!!  :rolleyes: 

111.

BĐT cần chứng minh tương đương

 

$(a+b+c)(\frac{a}{(b+c)^2}+\frac{b}{(c+a)^2}+\frac{c}{(a+b)^2})\geqslant \frac{9}{4}$

 

Áp dụng bđt Bunhiacopxki

 

$(a+b+c)(\frac{a}{(b+c)^2}+\frac{b}{(c+a)^2}+\frac{c}{(a+b)^2})\geqslant (\frac{a}{b+c}+\frac{b}{c+a}+\frac{c}{a+b})^2$

$\geqslant (\frac{3}{2})^2=\frac{9}{4}$ (đúng theo BĐT Nesbit)




#487348 $\boxed{\text{Chuyên Đề}}$ Bất đẳng thức - Cực trị

Posted by lahantaithe99 on 17-03-2014 - 13:50 in Bất đẳng thức và cực trị

Loạn hết rồi à? @@
 

106) Cho $a;b;c>0$ thỏa: $a^2+b^2+c^2=1$. Cmr: $\sum \frac{1}{a^2+b^2}\leq \frac{\sum a^3}{2abc}+3$

 

107)

a) Cho $x;y>0$. Cmr: $\sum \frac{1}{x^2}\geq \frac{8}{(x+y)^2}$

b) Cho $a;b;c>0$ thỏa: $ab+bc+ca\leq abc$. Cmr: $\sum \frac{8}{a+b}\leq \sum \frac{b+c}{a^2}+2$

 

 

 

 

106

Áp dụng bđt Co si $b^2+c^2\geqslant 2bc\Rightarrow \frac{a^2}{b^2+c^2}\leqslant \frac{a^2}{2bc}$

 

$\Rightarrow \sum \frac{a^2}{b^2+c^2}\leqslant \sum \frac{a^2}{2bc}=\frac{a^3+b^3+c^3}{2abc}$

 

$\Rightarrow \sum \frac{a^2+b^2+c^2}{b^2+c^2}\leqslant \frac{a^3+b^3+c^3}{2abc}+3$

$\Leftrightarrow \sum \frac{1}{b^2+c^2}\leqslant \frac{a^3+b^3+c^3}{2abc}+3$

Bài 107

a

$\sum \frac{1}{x^2}\geqslant \frac{1}{2}(\frac{1}{x}+\frac{1}{y})^2=\frac{1}{2}.\frac{(x+y)^2}{x^2y^2}$ $(1)$

 

Theo bdt Co si $xy\leqslant \frac{(x+y)^2}{4}\Rightarrow x^2y^2\leqslant \frac{(x+y)^4}{16}$ $(2)$

 

$(1);(2)\Rightarrow \sum \frac{1}{x^2}\geqslant \frac{8}{(x+y)^2}$

b) Từ phần a suy ra

 

$\frac{a+b}{a^2}+\frac{a+b}{b^2}\geqslant \frac{8}{a+b}$

 

Do đó

 

$\sum \frac{8}{a+b}\leqslant \frac{a+b}{a^2}+\frac{a+b}{b^2}+\frac{b+c}{b^2}+\frac{b+c}{c^2}+\frac{a+c}{a^2}+\frac{a+c}{c^2}$

 

$=\frac{a}{b^2}+\frac{b}{a^2}+\frac{c}{b^2}+\frac{b}{c^2}+\frac{c}{a^2}+\frac{a}{c^2}+2(\frac{1}{a}+\frac{1}{b}+\frac{1}{c})$

 

$=\frac{a+c}{b^2}+\frac{b+c}{a^2}+\frac{a+b}{c^2}+2(\frac{1}{a}+\frac{1}{b}+\frac{1}{c})$ $(1)$

 

Từ $GT :ab+bc+ac\leqslant abc\Rightarrow \frac{1}{a}+\frac{1}{b}+\frac{1}{c}\leqslant 1\Rightarrow 2\sum\frac{1}{a}\leqslant 2$ $(2)$

 

Từ $(1);(2)$ ta đc đpcm

c)

Áp dụng bđt $AM-GM$

$\frac{1}{(a-b)^2}+\frac{1}{(b-c)^2}\geqslant \frac{2}{(a-b)(b-c)}\geqslant \frac{8}{(a-c)^2}$

$\Rightarrow VT\geqslant \frac{a^2+c^2}{2}+\frac{8}{(a-c)^2}$

$=\frac{2ac}{2}+\frac{(a-c)^2}{2}+\frac{8}{(a-c)^2}\geqslant ac+4$ (cô si)




#483507 $\boxed{\text{Chuyên Đề}}$ Bất đẳng thức - Cực trị

Posted by lahantaithe99 on 16-02-2014 - 19:23 in Bất đẳng thức và cực trị

 

 

69) Cho $a;b;c>0$ thỏa: $12\sum \frac{1}{a^2}=3+\sum \frac{1}{a}$. Cmr: $\sum \frac{1}{4a+b+c}\leq \frac{1}{6}$

Áp dụng $AM-GM$

$\frac{1}{a^2}+\frac{1}{9}\geq \frac{2}{3a}$

$\Rightarrow \sum \frac{1}{a^2}\geq \sum \frac{2}{3a}-\frac{1}{3}$

$\Rightarrow \sum \frac{12}{a^2}\geq \sum \frac{8}{a}-4$

$\Rightarrow 3+\sum \frac{1}{a}\geq \sum \frac{8}{a}-4\Leftrightarrow \sum\frac{1}{a}\leq 1$

Áp dụng bđt S.Vacxơ

$\frac{1}{4a+b+c}\leq \frac{1}{36}(\frac{4}{a}+\frac{1}{b}+\frac{1}{c})$

CMTT $\sum \frac{1}{4a+b+c}\leq \frac{1}{36}(\frac{6}{a}+\frac{6}{b}+\frac{6}{c})\leq \frac{1}{6}$




#484449 $\boxed{\text{Chuyên Đề}}$ Bất đẳng thức - Cực trị

Posted by lahantaithe99 on 23-02-2014 - 20:22 in Bất đẳng thức và cực trị

 

84) Cho $\left\{\begin{matrix}a;b;c>0 & & \\ 3(ab+bc+ca)=1 & & \end{matrix}\right.$. Cmr: $\sum \frac{a}{a^2-bc+1}\geq \frac{1}{a+b+c}$

Lâu ròi ko lên diễn đàn :icon6:

$\sum \frac{a}{a^2-bc+1}=\sum \frac{a}{a^2+3ac+3ab+2bc}=\sum \frac{a^2}{a^3+3a^2c+3a^2b+2abc}$

$\geq \frac{(a+b+c)^2}{\sum a^3+3(a+b)(b+c)(c+a)}$ (áp dụng bđt S.Vac)

$= \frac{(a+b+c)^2}{(a+b+c)^3}=\frac{1}{a+b+c}$




#483502 $\boxed{\text{Chuyên Đề}}$ Bất đẳng thức - Cực trị

Posted by lahantaithe99 on 16-02-2014 - 19:06 in Bất đẳng thức và cực trị

 

68) Cho $a;b;c$ là 3 cạnh một tam giác có chu vi bằng $1$. Tìm Min $C=a^3+b^3+c^3+3abc$

 

 

Áp dụng tương tự bài 66 $3abc\geq \frac{4}{3}(ab+bc+ac)-\frac{1}{3}$

$\Rightarrow C\geq a^3+b^3+c^3+\frac{4}{3}(ab+bc+ac)-\frac{1}{3}$

Bằng $AM-GM$ có $a^3+b^3+c^3\geq\sum\frac{2a^2}{3}-\frac{1}{9}$

$\Rightarrow C\geq \sum\frac{2a^2}{3}+\frac{4}{3}(ab+bc+ac)-\frac{4}{9}=\frac{2}{3}-\frac{4}{9}=\frac{2}{9}$




#483495 $\boxed{\text{Chuyên Đề}}$ Bất đẳng thức - Cực trị

Posted by lahantaithe99 on 16-02-2014 - 18:26 in Bất đẳng thức và cực trị

66) Cho $a;b;c$ là 3 cạnh một tam giác có chu vi bằng $6$. Tìm Min $A=3(a^2+b^2+c^2)+2abc$
 

 

Ta có $abc\geq (a+b-c)(a+c-b)(b+c-a)=(6-2a)(6-2b)(6-2b)$

$=216-72(a+b+c)+24(ab+bc+ac)-8abc=24(ab+bc+ac)-216-8abc$

$\Rightarrow 9abc\geq 24(ab+bc+ac)-216\Leftrightarrow 2abc\geq \frac{16}{3}(ab+bc+ac)-48$

$\Rightarrow A\geq 3(a^2+b^2+c^2)+\frac{16}{3}(ab+bc+ac)-48$

$=\frac{a^2+b^2+c^2}{3}+\frac{8}{3}(a+b+c)^2-48$

$\geq 4+96-48=52$




#482960 $\boxed{\text{Chuyên Đề}}$ Bất đẳng thức - Cực trị

Posted by lahantaithe99 on 13-02-2014 - 20:49 in Bất đẳng thức và cực trị

 

60) Cho $a;b;c$ là độ dài 3 cạnh một tam giác. Cmr: $\sum \frac{a}{\sqrt{b+c-a}}\geq \sum \sqrt{a}$

 

 

 

P/s: Anh Daicagiangho1998 học KHTN nên cứ từ từ mà làm thôi, chứ có đề phát đã ăn hết sạch luôn vậy :D

Đặt $x^2=b+c-a;y^2=a+c-b;z^2=a+b-c$

$\Rightarrow a=\frac{y^2+z^2}{2};b=\frac{x^2+z^2}{2};c=\frac{x^2+y^2}{2}$

BĐT cần cm trở thành $\sum \frac{x^2+y^2}{2x}\geq \sum \sqrt{\frac{x^2+y^2}{2}}$

Áp dụng bđt Cauchy-Shwarz có

$\sum \frac{x^2+y^2}{2x}\geq \frac{(\sum\sqrt{\frac{x^2+y^2}{2}})^2 }{x+y+z}$

Bằng $AM-GM$ ta dễ chứng minh $\sum x\leq \sum \sqrt{\frac{x^2+y^2}{2}}$

$\Rightarrow \sum \frac{x^2+y^2}{2x}\geq \sum\sqrt{\frac{x^2+y^2}{2}}$ (đpcm)




#484618 $\boxed{\text{Chuyên Đề}}$ Bất đẳng thức - Cực trị

Posted by lahantaithe99 on 24-02-2014 - 18:59 in Bất đẳng thức và cực trị

91) Cho $x;y;z>0$ thỏa: $xyz\geq x+y+z+2$. Tìm Min $(x+y+z)$

94) Cho $a,b,c>0$ thỏa $ab+bc+ac\leq3abc$. Cmr $\sum\frac{a^4b}{2a+b}\geq1$

91

Có $\frac{(x+y+z)^3}{27}\geq xyz=x+y+z+2$

$\frac{(x+y+z)^3}{27}-(x+y+z+2)\geq 0$

Đặt $x+y+z=t\Rightarrow t^3-27t-54\geq 0\Rightarrow t\geq 6$

94

Ta có áp dụng Bunhiacopxki

$\sum \frac{a^4b}{2a+b}=\sum \frac{a^4b^2}{2ab+b^2}$

$\geq \frac{(a^2b+b^2c+c^2a)^2}{(a+b+c)^2}$

Lại có $(a^2b+b^2c+c^2a)(\frac{1}{a}+\frac{1}{b}+\frac{1}{c})\geq (a+b+c)^2$ (theo Bunhia)

$\Rightarrow (a^2b+b^2c+c^2a)(\frac{ab+bc+ac}{abc})\geq (a+b+c)^2$

$\Rightarrow (a^2b+b^2c+c^2a)\geq \frac{(a+b+c)^2}{3}$ (do $ab+bc+ac\leq3abc$)

$\Rightarrow (a^2b+b^2c+c^2a)^2\geq \frac{(a+b+c)^4}{9}$

Do đó $\sum \frac{a^4b}{2a+b}\geq \frac{(a+b+c)^4}{9(a+b+c)^2}=\frac{(a+b+c)^2}{9}$ $(1)$

Từ giả thiết $ab+bc+ac\leq 3abc\Rightarrow ab+bc+ac\geq 3\Rightarrow a+b+c\geq 3$ $(2)$

$(1);(2)\Rightarrow \sum \frac{a^4b}{2a+b}\geq 1$




#484933 $\boxed{\text{Chuyên Đề}}$ Bất đẳng thức - Cực trị

Posted by lahantaithe99 on 26-02-2014 - 20:32 in Bất đẳng thức và cực trị

96/ Cho $0<x;y;z \leq \frac{3}{2}$. Chứng minh rằng $\sum \sqrt{x^2+\frac{1}{x^2}} \geq \frac{3}{2}.\sqrt{17}$

 

 

Áp dụng bđt Bunhia

 

 

$\sqrt{(x^2+\frac{1}{x^2})(1+4^2)}\geq x+\frac{4}{x}$

 

$\Rightarrow \sqrt{17}.\sum \sqrt{x^2+\frac{1}{x^2}}\geq\sum x+\sum \frac{4}{x}$

 

$\geq\sum x+\sum \frac{1}{4x}+\sum \frac{15}{4x}$

 

$\geq 3+\sum \frac{15}{4x}$ (áp dụng cô si)

 

$\geq 3+\frac{15.9}{4x+4y+4z}\geq 3+\frac{45}{2}=\frac{51}{2}$

 

$\Rightarrow \sum \sqrt{x^2+\frac{1}{x^2}}\geq \frac{51}{2\sqrt{17}}=\frac{3}{2}.\sqrt{17}$




#486304 $\boxed{\text{Chuyên Đề}}$ Bất đẳng thức - Cực trị

Posted by lahantaithe99 on 08-03-2014 - 20:03 in Bất đẳng thức và cực trị

Đây nữa 

112/

Các số a,b,c dương  thỏa mãn abc=1. cmr:

S=$\frac{1}{(a+1)^2+b^2+1} +\frac{1}{(b+1)^2+c^2+1}+\frac{1}{(c+1)^2+a^2+1}$ $\leq$ $\frac{1}{2}$

Ta có $S=\sum \frac{1}{(a+1)^2+b^2+1}=\sum \frac{1}{a^2+2a+2+b^2}\leq \sum \frac{1}{2(ab+a+1)}$

 

Do đó $S\leq\frac{1}{2}(\frac{1}{ab+a+1}+\frac{1}{bc+b+1}+\frac{1}{ac+a+1})$

 

Từ giả thiết $abc=1$ ta dễ tính đc biểu thức trong ngoặc $=1$

 

suy ra $S\leq\frac{1}{2}$




#486118 $\boxed{\text{Chuyên Đề}}$ Bất đẳng thức - Cực trị

Posted by lahantaithe99 on 07-03-2014 - 12:36 in Bất đẳng thức và cực trị

Mình xin đóng góp ít bài :icon6: .

 

109) Cho a,b,c>0 t/m: $a+b+c\geq \frac{1}{a}+\frac{1}{b}+\frac{1}{c}$.

CMR: $a+b+c\geq \frac{3}{a+b+c}+\frac{2}{abc}$

 

Từ giả thiết dễ dàng suy ra $a+b+c\geq 3\Leftrightarrow \frac{a+b+c}{3}\geq \frac{3}{a+b+c}(1)$

$GT\Leftrightarrow abc(a+b+c)\geq ab+bc+ac\Rightarrow \frac{2}{abc}\leq \frac{2(a+b+c)}{ab+bc+ac}$

Lại có $\frac{(ab+bc+ac)^2}{3}\geq abc(a+b+c)\geq ab+bc+ac\Rightarrow ab+bc+ac\geq 3$

$\Rightarrow \frac{2}{abc}\leq \frac{2(a+b+c)}{ab+bc+ac}\leq \frac{2(a+b+c)}{3}(2)$

$(1);(2)$ suy ra đpcm




#485617 $\boxed{\text{Chuyên Đề}}$ Bất đẳng thức - Cực trị

Posted by lahantaithe99 on 03-03-2014 - 12:55 in Bất đẳng thức và cực trị

 

110) Cho a,b,c>0 t/m: a+b+c=1. Cmr: $a\sqrt[3]{1+b-c}+b\sqrt[3]{1+c-a}+c\sqrt[3]{1+a-b}\leq 1$

110

$VT=\sum a\sqrt[3]{1+b-c}=\sum a\sqrt[3]{a+2b}$

áp dụng bđt cô si ta có

$a\sqrt[3]{a+2b}\leq \frac{a(1+1+a+2b)}{3}=\frac{a^2+2ab+2a}{3}$

$\Rightarrow \sum a\sqrt[3]{a+2b}\leq \sum \frac{a^2+2ab+2a}{3}=\frac{(a+b+c)^2+2(a+b+c)}{3}=1$




#485492 $\boxed{\text{Chuyên Đề}}$ Bất đẳng thức - Cực trị

Posted by lahantaithe99 on 02-03-2014 - 17:27 in Bất đẳng thức và cực trị

103

$a\sqrt{ac}\leq a\frac{a+c}{2}$$\Rightarrow \sum a\sqrt{ac}\leq \frac{\sum a^{2}+\sum ab}{2}$

lại có $\sum \frac{a^{3}}{b}\sum \frac{a^{4}}{ab}\geq {(\sum a^{2})^{2}}{\sum ab}$

ta cần cm ${(\sum a^{2})^{2}}{\sum ab}\geq \frac{\sum a^{2}+\sum ab}{2}$

mà $\sum a^{2}\geq \sum ab$

$\Rightarrow (\sum a^{2})^{2}\geq (\sum a^{2})(\sum ab)$

$\Rightarrow (\sum a^{2})^{2}\geq (\sum ab)^{2}$

nên ta có đpcm

Chỗ này là sao hả Hoàng????




#482583 $\boxed{\text{Chuyên Đề}}$ Bất đẳng thức - Cực trị

Posted by lahantaithe99 on 11-02-2014 - 20:21 in Bất đẳng thức và cực trị

 

28) Cho $a;b;c>0$ thỏa $a+b+c=3$. Cmr: $\sum \frac{a+1}{b^2+1}\geq 3$

 

 

Ta có

$\frac{a+1}{b^2+1}=(a+1)-\frac{ab^2+b^2}{b^2+1}\geq (a+1)-\frac{ab+b}{2}$

$\Rightarrow \sum \frac{a+1}{b^2+1}\geq 6-\sum \frac{ab+b}{2}\geq 6-3=3$

do dễ cm $\sum \frac{ab+b}{2}\leq 3$ với $a+b+c=3$